6
$\begingroup$

Let $d(n)$ denote the number of divisors of $n$. Is it known that the series $$\sum_{p \text{ prime}} \frac{1}{d(p-1)}$$ diverges?

This would follow immediately from the Sophie Germain Conjecture. Indeed, if there are infinitely many primes of the form $2p+1$ ($p$ a prime), then infinitely many terms of the series are equal to $1/4$, so the series doesn't even satisfy the most basic requirement for convergence! So, surely there must be a direct proof?

$\endgroup$
3
  • $\begingroup$ See Igor's answer for an argument along the lines you want. mathoverflow.net/questions/27508/factors-of-p-1-when-p-is-prime $\endgroup$ Apr 25, 2013 at 5:43
  • 5
    $\begingroup$ FYI: Since $1/d(n)$ is a multiplicative function that is $1/2$ on every prime, standard heuristics predict that the summatory function $\sum_{n\le x} 1/d(n)$ should have order of magnitude $x(\log x)^{1/2-1}$. (One could probably even write down the leading constant if careful.) On the further heuristic that random shifted primes $p-1$ aren't all that different from random $n$, one predicts that $\sum_{p\le x} 1/d(p-1)$ should have order of magnitude $x(\log x)^{-3/2}$. $\endgroup$ Apr 25, 2013 at 7:52
  • $\begingroup$ Interesting, @Greg - thanks for sharing. $\endgroup$ Apr 27, 2013 at 22:36

3 Answers 3

9
$\begingroup$

Since every divisor $k$ of $p-1$ either satisfies $k\le \sqrt{p-1}$ or $\frac{p-1}{k}\le \sqrt{p-1}$, we have $d(p-1) \le 2\sqrt{p-1}$. If we let $p_n$ denote the $n$th prime, then since $p_n-1 \le n^2$ for all $n$ (an easy consequence of the prime number theroem...), we have

$\sum_{n=1}^N \frac{1}{d(p_n-1)} \ge \sum_{n=1}^N\frac{1}{2n} \ge \frac{\ln N}{2}$,

so the partial sums diverge.

Obviously one can get much better bounds than these.

$\endgroup$
4
  • $\begingroup$ Nitpick: the prime number theorem has as an easy consequence the fact that $p_n-1\le n^2$ for all sufficiently large $n$. $\endgroup$ Apr 25, 2013 at 7:41
  • 1
    $\begingroup$ Using $d(n)\ll n^{\epsilon}$ for any $\epsilon>0$, we obtain that the sum is $\gg N^{1-\epsilon}$. $\endgroup$ May 9, 2017 at 23:42
  • $\begingroup$ Using the normal order of the number of prime factors of $p-1$, we obtain that the sum is $\gg N/(\log N)^{1+\log 2 + \epsilon}$. I do agree that it might be in fact $\approx N/ (\log N)^{3/2}$. $\endgroup$ May 11, 2017 at 3:34
  • $\begingroup$ What I meant in the above comment was for the sum $\sum_{n\leq N} \frac1{d(p-1)}$. $\endgroup$ May 13, 2017 at 19:11
4
$\begingroup$

Answering my own question, because I totally overlooked the following ridiculous idea:

Obviously $d(n)\leq n$ for every $n$. Thus $d(p-1)\leq p-1 < p$, so $1/d(p-1) > 1/p$ and the divergence follows from the divergence of $\sum 1/p$ (if one is willing to assume that).

$\endgroup$
0
$\begingroup$

I did a heuristic analysis to study how this sum is growing. I calculated $d(p_n-1)$ and the sum and plotted the curve of the sum vs $n$. I obtained a very smooth curve which looked like a cumulative distribution curve. Next I did curve fitting to model this curve where the sum is the dependent variable and $n$ is the independent variable. The boundary condition I imposed was that the sum should be divergent. The following model was found to best fit the desired sum.

$$ \sum_{n=1}^{x}\frac{1}{d(p_n - 1)} \sim e^{a + b/x + c\ln\ln x} $$

where $a,b$ and $c$ are suitable constants.

The true asymptotic formula could be different from the above but I believe this can give some hints in the direction of the true asymptotics.

$\endgroup$
3
  • 2
    $\begingroup$ Presumably your $N$ and $x$ are the same. Note that $e^{a+b/x+c\ln x} = e^a x^c e^{b/x} \sim e^a x^c$. So your model is simply predicting that the sum is asymptotic to a constant times a power of $x$. This is almost surely false: the sum is bounded above by $cx/\log x$ by the prime number theorem, and bounded below by $cx/(\log x)^2$ if there are infinitely many Sophie Germaine primes. As I commented on the original post, the sum is probably asymptotic to a constant times $x(\log x)^{-3/2}$. $\endgroup$ Apr 28, 2013 at 2:38
  • $\begingroup$ @GregMartin The bound $x/\log x$ is from counting the number of primes up to $x$. Here in the question, the prime counting is already done by indexing $p_n$, and $n=1, \cdots , \lfloor x \rfloor$. Thus, the heuristic $e^a x$ would possibly makes sense. $\endgroup$ May 9, 2017 at 22:45
  • 1
    $\begingroup$ A fair point. I still think the conclusion is false, for reasons stated above. $\endgroup$ May 10, 2017 at 0:36

Your Answer

By clicking “Post Your Answer”, you agree to our terms of service and acknowledge you have read our privacy policy.

Not the answer you're looking for? Browse other questions tagged or ask your own question.